Are $sigma$-finite measures agreeing on a generating set equal?












0












$begingroup$


Suppose I have two measures $m,n$ and that they are both $sigma$-finite and agree on some semi-ring $R$ generating their $sigma$-algebras. Must these two measures then agree on the entire $sigma$-algebra?



Of course, the Caratheodory extension theorem states that if the two pre-measures given by the restriction of $m,n$ to $R$ are themselves $sigma$-finite, then their extension to the $sigma$-algebra is unique, but I suspect there might be a case where the restriction of a $sigma$-finite measure to the semi-ring might not remain $sigma$-finite.










share|cite|improve this question











$endgroup$












  • $begingroup$
    Let $mathcal{B}$ be the $sigma$-algebra. Then ${A in mathcal{B} : m(A) = n(A)}$ is a sigma-algebra containing $R$ and is thus $mathcal{B}$.
    $endgroup$
    – mathworker21
    Dec 1 '18 at 13:58










  • $begingroup$
    Don't we run into a problem because the measures may not agree on complements of sets of infinite measure?
    $endgroup$
    – Bar Alon
    Dec 1 '18 at 14:23
















0












$begingroup$


Suppose I have two measures $m,n$ and that they are both $sigma$-finite and agree on some semi-ring $R$ generating their $sigma$-algebras. Must these two measures then agree on the entire $sigma$-algebra?



Of course, the Caratheodory extension theorem states that if the two pre-measures given by the restriction of $m,n$ to $R$ are themselves $sigma$-finite, then their extension to the $sigma$-algebra is unique, but I suspect there might be a case where the restriction of a $sigma$-finite measure to the semi-ring might not remain $sigma$-finite.










share|cite|improve this question











$endgroup$












  • $begingroup$
    Let $mathcal{B}$ be the $sigma$-algebra. Then ${A in mathcal{B} : m(A) = n(A)}$ is a sigma-algebra containing $R$ and is thus $mathcal{B}$.
    $endgroup$
    – mathworker21
    Dec 1 '18 at 13:58










  • $begingroup$
    Don't we run into a problem because the measures may not agree on complements of sets of infinite measure?
    $endgroup$
    – Bar Alon
    Dec 1 '18 at 14:23














0












0








0


2



$begingroup$


Suppose I have two measures $m,n$ and that they are both $sigma$-finite and agree on some semi-ring $R$ generating their $sigma$-algebras. Must these two measures then agree on the entire $sigma$-algebra?



Of course, the Caratheodory extension theorem states that if the two pre-measures given by the restriction of $m,n$ to $R$ are themselves $sigma$-finite, then their extension to the $sigma$-algebra is unique, but I suspect there might be a case where the restriction of a $sigma$-finite measure to the semi-ring might not remain $sigma$-finite.










share|cite|improve this question











$endgroup$




Suppose I have two measures $m,n$ and that they are both $sigma$-finite and agree on some semi-ring $R$ generating their $sigma$-algebras. Must these two measures then agree on the entire $sigma$-algebra?



Of course, the Caratheodory extension theorem states that if the two pre-measures given by the restriction of $m,n$ to $R$ are themselves $sigma$-finite, then their extension to the $sigma$-algebra is unique, but I suspect there might be a case where the restriction of a $sigma$-finite measure to the semi-ring might not remain $sigma$-finite.







real-analysis measure-theory






share|cite|improve this question















share|cite|improve this question













share|cite|improve this question




share|cite|improve this question








edited Dec 2 '18 at 15:29







Bar Alon

















asked Dec 1 '18 at 13:53









Bar AlonBar Alon

484114




484114












  • $begingroup$
    Let $mathcal{B}$ be the $sigma$-algebra. Then ${A in mathcal{B} : m(A) = n(A)}$ is a sigma-algebra containing $R$ and is thus $mathcal{B}$.
    $endgroup$
    – mathworker21
    Dec 1 '18 at 13:58










  • $begingroup$
    Don't we run into a problem because the measures may not agree on complements of sets of infinite measure?
    $endgroup$
    – Bar Alon
    Dec 1 '18 at 14:23


















  • $begingroup$
    Let $mathcal{B}$ be the $sigma$-algebra. Then ${A in mathcal{B} : m(A) = n(A)}$ is a sigma-algebra containing $R$ and is thus $mathcal{B}$.
    $endgroup$
    – mathworker21
    Dec 1 '18 at 13:58










  • $begingroup$
    Don't we run into a problem because the measures may not agree on complements of sets of infinite measure?
    $endgroup$
    – Bar Alon
    Dec 1 '18 at 14:23
















$begingroup$
Let $mathcal{B}$ be the $sigma$-algebra. Then ${A in mathcal{B} : m(A) = n(A)}$ is a sigma-algebra containing $R$ and is thus $mathcal{B}$.
$endgroup$
– mathworker21
Dec 1 '18 at 13:58




$begingroup$
Let $mathcal{B}$ be the $sigma$-algebra. Then ${A in mathcal{B} : m(A) = n(A)}$ is a sigma-algebra containing $R$ and is thus $mathcal{B}$.
$endgroup$
– mathworker21
Dec 1 '18 at 13:58












$begingroup$
Don't we run into a problem because the measures may not agree on complements of sets of infinite measure?
$endgroup$
– Bar Alon
Dec 1 '18 at 14:23




$begingroup$
Don't we run into a problem because the measures may not agree on complements of sets of infinite measure?
$endgroup$
– Bar Alon
Dec 1 '18 at 14:23










1 Answer
1






active

oldest

votes


















0












$begingroup$

It turns out that the answer is no.



Consider the two measures $m,n$ defined on the borel $sigma$-algebra $mathcal{B}(mathbb{R})$ as follows:
$$m(A)=|Acapmathbb{Q}|,quad n(A)=|Acap(mathbb{Q}cup{sqrt{2}})|$$
Since $mathbb{Q}$ is countable, they are both $sigma$-finite. Additionally, they agree on the semi-ring $R$ of all half-open intervals (for any $a,bin mathbb{R}$: $m([a,b))=infty=n([a,b))$), and $R$ indeed generates $mathcal{B}(mathbb{R})$.



On the other hand $m({sqrt{2}})not =n({sqrt{2}})$ which is clearly a borel set.



Note that in the above example $m, n$ are indeed no longer $sigma$-finite when restricted to $R$.






share|cite|improve this answer









$endgroup$













    Your Answer





    StackExchange.ifUsing("editor", function () {
    return StackExchange.using("mathjaxEditing", function () {
    StackExchange.MarkdownEditor.creationCallbacks.add(function (editor, postfix) {
    StackExchange.mathjaxEditing.prepareWmdForMathJax(editor, postfix, [["$", "$"], ["\\(","\\)"]]);
    });
    });
    }, "mathjax-editing");

    StackExchange.ready(function() {
    var channelOptions = {
    tags: "".split(" "),
    id: "69"
    };
    initTagRenderer("".split(" "), "".split(" "), channelOptions);

    StackExchange.using("externalEditor", function() {
    // Have to fire editor after snippets, if snippets enabled
    if (StackExchange.settings.snippets.snippetsEnabled) {
    StackExchange.using("snippets", function() {
    createEditor();
    });
    }
    else {
    createEditor();
    }
    });

    function createEditor() {
    StackExchange.prepareEditor({
    heartbeatType: 'answer',
    autoActivateHeartbeat: false,
    convertImagesToLinks: true,
    noModals: true,
    showLowRepImageUploadWarning: true,
    reputationToPostImages: 10,
    bindNavPrevention: true,
    postfix: "",
    imageUploader: {
    brandingHtml: "Powered by u003ca class="icon-imgur-white" href="https://imgur.com/"u003eu003c/au003e",
    contentPolicyHtml: "User contributions licensed under u003ca href="https://creativecommons.org/licenses/by-sa/3.0/"u003ecc by-sa 3.0 with attribution requiredu003c/au003e u003ca href="https://stackoverflow.com/legal/content-policy"u003e(content policy)u003c/au003e",
    allowUrls: true
    },
    noCode: true, onDemand: true,
    discardSelector: ".discard-answer"
    ,immediatelyShowMarkdownHelp:true
    });


    }
    });














    draft saved

    draft discarded


















    StackExchange.ready(
    function () {
    StackExchange.openid.initPostLogin('.new-post-login', 'https%3a%2f%2fmath.stackexchange.com%2fquestions%2f3021368%2fare-sigma-finite-measures-agreeing-on-a-generating-set-equal%23new-answer', 'question_page');
    }
    );

    Post as a guest















    Required, but never shown

























    1 Answer
    1






    active

    oldest

    votes








    1 Answer
    1






    active

    oldest

    votes









    active

    oldest

    votes






    active

    oldest

    votes









    0












    $begingroup$

    It turns out that the answer is no.



    Consider the two measures $m,n$ defined on the borel $sigma$-algebra $mathcal{B}(mathbb{R})$ as follows:
    $$m(A)=|Acapmathbb{Q}|,quad n(A)=|Acap(mathbb{Q}cup{sqrt{2}})|$$
    Since $mathbb{Q}$ is countable, they are both $sigma$-finite. Additionally, they agree on the semi-ring $R$ of all half-open intervals (for any $a,bin mathbb{R}$: $m([a,b))=infty=n([a,b))$), and $R$ indeed generates $mathcal{B}(mathbb{R})$.



    On the other hand $m({sqrt{2}})not =n({sqrt{2}})$ which is clearly a borel set.



    Note that in the above example $m, n$ are indeed no longer $sigma$-finite when restricted to $R$.






    share|cite|improve this answer









    $endgroup$


















      0












      $begingroup$

      It turns out that the answer is no.



      Consider the two measures $m,n$ defined on the borel $sigma$-algebra $mathcal{B}(mathbb{R})$ as follows:
      $$m(A)=|Acapmathbb{Q}|,quad n(A)=|Acap(mathbb{Q}cup{sqrt{2}})|$$
      Since $mathbb{Q}$ is countable, they are both $sigma$-finite. Additionally, they agree on the semi-ring $R$ of all half-open intervals (for any $a,bin mathbb{R}$: $m([a,b))=infty=n([a,b))$), and $R$ indeed generates $mathcal{B}(mathbb{R})$.



      On the other hand $m({sqrt{2}})not =n({sqrt{2}})$ which is clearly a borel set.



      Note that in the above example $m, n$ are indeed no longer $sigma$-finite when restricted to $R$.






      share|cite|improve this answer









      $endgroup$
















        0












        0








        0





        $begingroup$

        It turns out that the answer is no.



        Consider the two measures $m,n$ defined on the borel $sigma$-algebra $mathcal{B}(mathbb{R})$ as follows:
        $$m(A)=|Acapmathbb{Q}|,quad n(A)=|Acap(mathbb{Q}cup{sqrt{2}})|$$
        Since $mathbb{Q}$ is countable, they are both $sigma$-finite. Additionally, they agree on the semi-ring $R$ of all half-open intervals (for any $a,bin mathbb{R}$: $m([a,b))=infty=n([a,b))$), and $R$ indeed generates $mathcal{B}(mathbb{R})$.



        On the other hand $m({sqrt{2}})not =n({sqrt{2}})$ which is clearly a borel set.



        Note that in the above example $m, n$ are indeed no longer $sigma$-finite when restricted to $R$.






        share|cite|improve this answer









        $endgroup$



        It turns out that the answer is no.



        Consider the two measures $m,n$ defined on the borel $sigma$-algebra $mathcal{B}(mathbb{R})$ as follows:
        $$m(A)=|Acapmathbb{Q}|,quad n(A)=|Acap(mathbb{Q}cup{sqrt{2}})|$$
        Since $mathbb{Q}$ is countable, they are both $sigma$-finite. Additionally, they agree on the semi-ring $R$ of all half-open intervals (for any $a,bin mathbb{R}$: $m([a,b))=infty=n([a,b))$), and $R$ indeed generates $mathcal{B}(mathbb{R})$.



        On the other hand $m({sqrt{2}})not =n({sqrt{2}})$ which is clearly a borel set.



        Note that in the above example $m, n$ are indeed no longer $sigma$-finite when restricted to $R$.







        share|cite|improve this answer












        share|cite|improve this answer



        share|cite|improve this answer










        answered Dec 2 '18 at 15:27









        Bar AlonBar Alon

        484114




        484114






























            draft saved

            draft discarded




















































            Thanks for contributing an answer to Mathematics Stack Exchange!


            • Please be sure to answer the question. Provide details and share your research!

            But avoid



            • Asking for help, clarification, or responding to other answers.

            • Making statements based on opinion; back them up with references or personal experience.


            Use MathJax to format equations. MathJax reference.


            To learn more, see our tips on writing great answers.




            draft saved


            draft discarded














            StackExchange.ready(
            function () {
            StackExchange.openid.initPostLogin('.new-post-login', 'https%3a%2f%2fmath.stackexchange.com%2fquestions%2f3021368%2fare-sigma-finite-measures-agreeing-on-a-generating-set-equal%23new-answer', 'question_page');
            }
            );

            Post as a guest















            Required, but never shown





















































            Required, but never shown














            Required, but never shown












            Required, but never shown







            Required, but never shown

































            Required, but never shown














            Required, but never shown












            Required, but never shown







            Required, but never shown







            Popular posts from this blog

            Quarter-circle Tiles

            build a pushdown automaton that recognizes the reverse language of a given pushdown automaton?

            Mont Emei